Your-Doctor
Multiple Choice Questions (MCQ)


Quiz Categories Click to expand

Category: Cardiology--->Adult Congenital Heart Disease and Pregnancy
Page: 4

Question 16# Print Question

Which one of the following patients would you advise to avoid pregnancy?

A. A 25-year-old with repaired ToF and severe pulmonary regurgitation with a mildly dilated and mildly impaired right ventricle
B. A 32-year-old with idiopathic pulmonary arterial hypertension, which has responded well to bosentan, who has a right atrial to right ventricular pressure drop of 45 mmHg
C. A 39-year-old patient with moderate mitral regurgitation and good left ventricular function
D. An 18-year-old with a single-ventricle circulation and a total cavopulmonary connection operation for tricuspid atresia, normally saturated and with good LV function
E. A 28-year-old with a Senning repair of the transposition of the great arteries with mildly reduced systemic ventricular function and mild to moderate tricuspid regurgitation


Question 17# Print Question

A 36-year-old patient with repaired ToF wishes to become pregnant and asks you about the likelihood of her child being born with a congenital heart defect. She has no family history of congenital heart disease. 

What is the approximate risk of her child having congenital heart disease?

A. 1–2%
B. 8–10%
C. 50%
D. 4–5%
E. No more than the general population. Most cases are sporadic


Question 18# Print Question

A GP writes to you to ask which contraceptive is advisable for her 35-year-old patient with a mechanical mitral valve replacement. She has had one child and several miscarriages because of taking warfarin. She does not wish to become pregnant again. 

What is the best method of contraception for this patient?

A. Sterilization
B. Condoms
C. Mirena intra-uterine system
D. Depo Provera
E. Combined oral contraceptive pill


Question 19# Print Question

 In current regulations, which of the following drugs is absolutely contraindicated in pregnancy?

A. Atenolol
B. Simvastatin
C. Aspirin
D. Amlodipine
E. Clopidogrel


Question 20# Print Question

A 35-year-old woman with a history of atrioventricular nodal tachycardia presents in premature labour at 36 weeks with a narrow complex tachycardia at 180 bpm (see ECG below).

What is the most appropriate drug to use after vagal manoeuvres and adenosine?

A. Esmolol
B. Amiodarone
C. Verapamil
D. Digoxin
E. Flecainide




Category: Cardiology--->Adult Congenital Heart Disease and Pregnancy
Page: 4 of 20